Đến nội dung

Ispectorgadget

Ispectorgadget

Đăng ký: 26-02-2011
Offline Đăng nhập: 16-04-2024 - 23:58
****-

#653461 Thăm dò ý kiến về việc thi trắc nghiệm môn toán

Gửi bởi Ispectorgadget trong 09-09-2016 - 12:27

:)) Sắp tới chắc thầy cô kéo nhau mở lớp dạy thêm toán Casio.




#651614 Tìm số nghiệm nguyên của hệ phương trình:

Gửi bởi Ispectorgadget trong 28-08-2016 - 10:22

Tìm số nghiệm nguyên của hệ phương trình: 

$$\left\{ \begin{array}{l}{x_1} + {x_2} + {x_3} + {x_4} + {x_5} = 25\\1 \le {x_i} \le 6,i \in \left\{ {1,2,3,4,5} \right\}\end{array} \right.$$

Spoiler

 

Đặt $y_i=x_i-1;\forall i=\overline{1,5}$. Từ giả thiết suy ra $0\le y_i\le 5$
Ta có hệ
$$(I) \; \left\{\begin{matrix}y_1+y_2+...+y_5=20\;\;\;\; (1)\\ 0\le y_i\le 5\; ; \forall i=\overline{1,5}\end{matrix}\right.$$
 
Gọi |X| là tập các nghiệm nguyên không âm của phương trình (1) ta có $|X|=C_{24}^4$.
Gọi $|A|,|B|,|C|,|D|,|E|$ lần lượt là tập tất cả các nghiệm của 5 hệ
$$\left\{\begin{matrix}y_1+y_2+...+y_5=20\\ y_i\ge 6\; ;\forall i\in \{1,2,3,4,5\}\end{matrix}\right.$$
Bằng cách đặt $k_i=y_i-6$ và áp dụng bài toán chia kẹo Euler ta dễ dàng tính được 
$|A|=|B|=|C|=|D|=|E|=0$ (phương trình này vô nghiệm :)) thấy lạ lạ ...)
 
$$|A\cap B| = |A\cap C| = |A\cap D| = |A\cap E| = |B\cap C| = |B\cap D| = |B\cap E| = |C\cap D| = |C\cap E| = |D\cap E| =0$$
$$|A\cap B\cap C| = |A\cap B\cap  D| = |A\cap B \cap E| = |A\cap C\cap D| = |A\cap C\cap E| = |A\cap D\cap E| = |B\cap C\cap D| =0$$
$|B\cap C\cap E| + |B\cap D\cap E| + |C\cap D\cap E| =0$
$$|A\cap B\cap C\cap D| = |A\cap B\cap C\cap E| = |A\cap B\cap D\cap E| = |A\cap C\cap D\cap E| = |B\cap C\cap D\cap E| =0$$
$$ |A\cap B\cap C\cap D\cap E| =0$$
 
Theo nguyên lý bù trừ ta có số nghiệm hệ (I) là
$$X-(|A| + |B| + |C| + |D| + |E| - |A\cap B| - |A\cap C| - |A\cap D| - |A\cap E| - |B\cap C| - |B\cap D| - |B\cap E| - |C\cap D| - |C\cap E| - |D\cap E| + |A\cap B\cap C| + |A\cap B\cap D| + |A\cap B\cap E| + |A\cap C\cap D| + |A\cap C\cap E| + |A\cap D\cap E| + |B\cap C\cap D| + |B\cap C\cap E| + |B\cap D\cap E| + |C\cap D\cap E| - |A\cap B\cap C\cap D| - |A\cap B\cap C\cap E| - |A\cap B\cap D\cap E| - |A\cap C\cap D\cap E| - |B\cap C\cap D\cap E| + |A\cap B\cap C\cap D\cap E| )$$
$$=C_{24}^4=10626. \;\; \blacksquare$$



#648077 CM vành con $A$ của vành $X$ là idean của X khi và chỉ kh...

Gửi bởi Ispectorgadget trong 05-08-2016 - 17:53

Chứng minh rằng một vành con $A$ của vành $X$ là idean của X khi và chỉ khi $A$ là hạt nhân của một đồng cấu $f:X\to Y$, với $Y$ là một vành nào đó.




#648076 Chứng minh $ab$ và $ba$ có cùng cấp; $a$ và...

Gửi bởi Ispectorgadget trong 05-08-2016 - 17:51

Cho $a,b$ là hai phần tử tùy ý của nhóm X. Chứng minh rằng:
1) $ab$ và $ba$ có cùng cấp.
2) $a$ và $b^{-1}ab$ có cùng cấp.



#645005 Thảo luận về Đề thi và Lời giải của IMO 2016

Gửi bởi Ispectorgadget trong 14-07-2016 - 22:40

Nguồn FB :)) 

File gửi kèm

  • File gửi kèm  imo.jpg   72K   75 Số lần tải
  • File gửi kèm  ket qua.jpg   38.07K   76 Số lần tải



#641033 $\sqrt[3]{cos\frac{2\pi}{7}...

Gửi bởi Ispectorgadget trong 18-06-2016 - 11:35

Chứng minh rằng:

$\sqrt[3]{cos\frac{2\pi}{7}}+\sqrt[3]{cos\frac{4\pi}{7}}+\sqrt[3]{cos\frac{8\pi}{7}}=\sqrt[3]{\frac{5-3\sqrt[3]{7}}{2}}$

Xét phương trình $\cos 4x=\cos 3x \Leftrightarrow (\cos x-1)(8\cos^3x+4\cos^2x-4\cos x-1)=0$

$\Leftrightarrow \cos x=1\vee 8\cos^3x+4\cos^2x-4\cos x-1=0$
Nhận thấy $t_1=2\cos \frac{2\pi}{7};t_2=2\cos \frac{4\pi}{7};t_3=2\cos\frac{6\pi}{7}$ là nghiệm của phương trình $t^3+t^2-2t-1=0$
Theo định lý Viet ta có \[\left\{ \begin{array}{l}
{t_1} + {t_2} + {t_3} = - 1\\
{t_1}{t_2} + {t_3}{t_2} + {t_1}{t_3} = - 2\\
{t_1}{t_2}{t_3} = 1
\end{array} \right.\]
Đặt $A=\sqrt[3]{t_1}+\sqrt[3]{t_2}+\sqrt[3]{t_3}$
$B=\sqrt[3]{t_1t_2}+\sqrt[3]{t_3t_2}+\sqrt[3]{t_1t_3}$
Ta có $A^3=3AB-4$ và $B^3=3AB-5$
$\Rightarrow A^3B^3=(3AB-4)(3AB-5)\Rightarrow (AB-3)^3+7=0\Rightarrow AB=3-\sqrt[3]{7}$
$\Rightarrow A^3=5-3\sqrt[3]{7}\Rightarrow A=\sqrt[3]{5-3\sqrt[3]{7}}$

Từ đây ta có đpcm. 




#639865 [Archive] Cập nhật list Những bài toán trong tuần (301-400)

Gửi bởi Ispectorgadget trong 12-06-2016 - 17:31

$\boxed{\text{Bài toán 359}}$

Cho đường tròn $(O)$.Hai đường tròn $(C1)$ và $(C2)$ tiếp xúc trong với $(O)$ lần lượt tại $A$ và $F$.Hai đường tròn này cắt nhau tại 2 điểm D và E phân biệt.Gọi K là hình chiếu của A lên DE.H là trung điểm của AK.M là trung điểm $DE$.Chứng minh góc $HMK$ bằng $\frac{1}{2}$ số đo cung nhỏ $AF$ của $(O)$.

 

$\boxed{\text{Bài toán 359}}$

Một tứ giác lồi được chia bởi các đường chéo thành 4 tam giác. Chứng minh rằng: đường thẳng nối các trọng tâm của 2 tam giác đối nhau vuông góc với đường thẳng nối các trực tâm của 2 tam giác còn lại. 

 

$\boxed{\text{Bài toán 360}}$

Tìm tất cả các số thực $x$ sao cho : $\cos(\cos(\cos(\cos x))))=\sin(\sin(\sin(\sin x)))$

 

$\boxed{\text{Bài toán 361}}$

Cho $a_1;a_2;...;a_n$ là dãy các số nguyên không âm. Với $k=1,2,....,n$,đặt $ m_k =\max_{1\le l\le k}\frac{a_{k-l+1}+a_{k-l+2}+\cdots+a_k}{l}. $

Chứng minh rằng với mỗi $\alpha>0$,số giá trị của $k$ thỏa mãn $m_k>\alpha$ luôn bé hơn $\frac{a_1+a_2+...+a_n}{\alpha}$

 

$\boxed{\text{Bài toán 362}}$

Cho bảng $8\times 6$,các ô của bảng được tô bởi $n$ màu sao cho mỗi cặp 2 màu chỉ xuất hiện cùng nhau không quá một hàng. Tìm giá trị nhỏ nhất có thể của $n$?

 

$\boxed{\text{Bài toán 363}}$

Tìm k để phương trình sau có 3 nghiệm phân biệt:

$$4^{-|x-k|}\log_{\sqrt{2}}(x^{2}-2x+3)+2^{-x^{2}+2x}\log_{\frac{1}{2}}(2|x-k|+2)=0$$

 

$\boxed{\text{Bài toán 364}}$

Tính tổng
$$S = \sum\limits_{n = 1}^5 {\sum\limits_{k = n}^{n + 4} {\left( {k.C_{k - 1}^{n - 1} .C_{9 - k}^{5 - n} } \right)} }$$

 

$\boxed{\text{Bài toán 365}}$

Cho $x$, $y$ là các số hữu tỉ thoả mãn đẳng thức $x^2+y^2+\left ( \frac{xy+1}{x+y} \right )^2 = 2$. Chứng minh rằng $\sqrt{1+xy}$ là một số hữu tỉ.

 

$\boxed{\text{Bài toán 366}}$

Chứng minh rằng với mọi số nguyên dương $k$ ; ta có đẳng thức :

$$\frac{1}{\sin^{2} \frac{\pi}{4k+2}} + \frac{1}{\sin^{2} \frac{3\pi}{4k+2}} + \frac{1}{\sin^{2} \frac{5\pi}{4k+2}}+ \cdots+ \frac{1}{\sin^{2} \frac{(2k-1)\pi}{4k+2}} = 2k(k+1)$$

 

$\boxed{\text{Bài toán 367}}$

Cho $p>2$ là 1 số nguyên tố. Chứng minh rằng phương trình:

$ax^2+by^2=pz+c$.

Có nghiệm $(x,y,z)\in N$, với $a,b,c\in N$ và không chia hết cho $p$.

 

 

$\boxed{\text{Bài toán 368}}$

Tính: $$\lim_{n\to \infty}\left ( \frac{1}{n}\sum_{k=1}^{n}\sqrt[k]{k} \right )$$




#623564 [Archive] Cập nhật list Những bài toán trong tuần (301-400)

Gửi bởi Ispectorgadget trong 30-03-2016 - 10:21

$\boxed{\text{Bài toán 348}}$

Cho $\Delta ABC$ trên tia đối tia $AB, BC, CA$ lần lượt vẽ các đoạn thẳng $AD, BE, CF$ sao cho $AB + AD = BC + BE = CA + CF$ ( hay $BD = CE = AF$ ). Chứng minh rằng : Nếu $\Delta DEF$ đều thì $\Delta ABC$ đều.

 

$\boxed{\text{Bài toán 349}}$

Giả sử phương trình $ax^2+bx+c=0(a \neq 0)$ có 2 nghiệm phân biệt.Xét dãy $\{x_{n} \}:\left\{\begin{matrix} x_0=\alpha \\ x_{n}(ax_{n-1}+b)+c=0;\forall n \in \mathbb{N^*} \end{matrix}\right.$.Tính $\lim x_{n}$ theo $\alpha$.

 

$\boxed{\text{Bài toán 350}}$

Tìm tất cả các hàm $f: \mathbb{N}\rightarrow \mathbb{N}$ thỏa:$$f(f(x)+x)+f(f(x)-x)=8x.$$

 

Các học sinh được phát bài kiểm tra , mỗi môn một bài, trong $n$ môn $(n\ge 3)$ môn học. Biết rằng với một môn học bất kì thì có đúng 3 học sinh đạt điểm tối ưu, còn với 2 môn tùy ý thì có 1 học sinh đạt điểm tối ưu cho mỗi môn trong cả 2 môn đó. Hãy xác định số $n$ bé nhất sao cho từ các điều kiện trên ta có thể suy ra rằng có đúng 1 học sinh đạt điểm tối ưu cho mỗi môn học trong $n$ môn đó.
 
Tìm tất cả các số tự nhiên n sao cho:
$$F_{n}(a,b,c)= a^n(b-c)+b^n(c-a)+c^n(a-b) \vdots (a^2+b^2+c^2+ab+bc+ca)$$
 
Chứng minh rằng với mọi số n nguyên dương có :

$$\frac{1!2!+2!3!+...+n!(n+1)!}{n\sqrt[n]{(1!)^2.(2!)^2...(n!)^2}}\geq 2\sqrt[2n]{n!}$$

 

$\boxed{\text{Bài toán 354}}$

Cho tứ giác $ABCD$ nội tiếp đường tròn tâm $O$ và ngoại tiếp đường tròn tâm $I$. $G$ là giao điểm hai đường chéo. Chứng minh rằng $O, I, G$ thằng hàng. Tổng quát bài toán.

 

$\boxed{\text{Bài toán 355}}$

Giải phương trình: $$(\sqrt{7-x^2}-2)(x^2-1)+x^2+(x-1)^2=2$$

 

$\boxed{\text{Bài toán 356}}$

Cho $a_1,a_2,a_3,a_4,a_5>0$. Chứng minh rằng:
$\frac{a_1+\sqrt{a_1a_2}+\sqrt[3]{a_1a_2a_3}+\sqrt[4]{a_1a_2a_3a_4}+\sqrt[5]{a_1a_2a_3a_4a_5}}{5} \leq \sqrt[5]{a_1.\frac{a_1+a_2}{2}.\frac{a_1+a_2+a_3}{3}.\frac{a_1+a_2+a_3+a_4}{4}.\frac{a_1+a_2+a_3+a_4+a_5}{5}}$

 

$\boxed{\text{Bài toán 357}}$

Giải bất phương trình:

$$25x^{4}+5x^{2}+9x(x^{2}+1)\sqrt{9x^{2}-4}-2\geq 0$$

 

$\boxed{\text{Bài toán 358}}$

Cho $m$ là số nguyên dương và $r$ là số thực ($r \geq 1$). Chứng minh:
$$\dfrac{1}{4rm} \left(\dfrac{(r + 1)^{r + 1}}{r^r}\right)^m < {(r + 1)m \choose m} < \left(\dfrac{(r + 1)^{r + 1}}{r^r}\right)^m$$
(với $z$ là số thực thì ${z \choose m}$ biểu thị $\dfrac{1}{m!}\prod_{k = 0}^{m - 1} (z - k)$.)




#621085 Cho $x,y$ thỏa $x^2+y^2xy=1$. Tìm min, max của $S=x...

Gửi bởi Ispectorgadget trong 19-03-2016 - 00:30

Cho $x,y$ thỏa $x^2+y^2xy=1$. Tìm min, max của 
$S=x^2+2xy-y^2$



#616361 Cho dãy số $(u_n)$ thỏa $u_1=1;u_{n+1}=2+\sqrt...

Gửi bởi Ispectorgadget trong 21-02-2016 - 23:44

Cho dãy số $(u_n)$ thỏa $u_1=1;u_{n+1}=2+\sqrt{u_n}-2\sqrt{\sqrt{u_n}+1}$
Xác định số hạng tổng quát $u_n$.



#615144 Tính tích phân $\int_0^\infty \frac{\sin x...

Gửi bởi Ispectorgadget trong 15-02-2016 - 12:21

Tính tích phân \[\int_0^\infty \frac{\sin x}{x(1+x)^2}\,dx\]




#613453 Rút gọn $E=\tan a. \tan 2a+\tan 2a. \tan 3a+...+...

Gửi bởi Ispectorgadget trong 07-02-2016 - 12:40

Rút gọn 
$E=\tan a. \tan 2a+\tan 2a. \tan 3a+...+\tan (n-1).\tan na$



#612486 Chứng minh rằng: Mỗi ma trận $B$ sao cho $AB=BA$ có dạng...

Gửi bởi Ispectorgadget trong 02-02-2016 - 19:44

Cho ma trận $A=\begin{pmatrix}2 &-1  &0 \\  -1&2  &-1 \\  0&-1  &2 \end{pmatrix}$

Chứng minh rằng: Mỗi ma trận $B$ sao cho $AB=BA$ có dạng
$$B=aI+bA+cA^2,$$
Với a,b,c là các số thực nào đó.



#609854 [Archive] Cập nhật list Những bài toán trong tuần (301-400)

Gửi bởi Ispectorgadget trong 19-01-2016 - 19:51

$\boxed{\text{Bài toán 333}}$ 

 Cho 1 tam giác đều được chia thành $n^2$ tam giác đều bằng nhau. Một trong số đó được đánh số bởi  1,2,3,…,m sao cho các tam giác với các số liên  tiếp phải có cạnh chung. Chứng minh rằng: $m\geq n^2-n+1$

 

$\boxed{\text{Bài toán 334}}$ 

Xét dãy $\{F_{n} \}_{n \ge 1}$ là dãy Fibonacci.Chứng minh đẳng thức Catalan:

$$F_{n}^2-F_{n+k}F_{n-k}=(-1)^{n-k}F_{k}^2(1 \le k \le n)$$

 

$\boxed{\text{Bài toán 335}}$ 

Cho $P,Q,R$ là các đa thức phức thỏa mãn
$$P^a+Q^b=R^c$$
Với $a,b,c$ là các số tự nhiên.
Chứng minh rằng:$\frac{1}{a}+\frac{1}{b}+\frac{1}{c} >1$

 

$\boxed{\text{Bài toán 336}}$ 

Cho tam giác $ABC$, các đường phân giác trong $BE,CI$. Chứng minh đẳng thức sau: 

$IE^2=\frac{bca^2}{(a+b)(a+c)}-2p(b-c)^2.\frac{abc}{(a+b)^2(a+c)^2}$
với $(p=\frac{\sum a}{2})$

 

 
Tìm tất cả các giá trị thực của $\alpha$ để cho $tan(\frac{5\pi }{12}+\alpha)$ là số hạng giửa của cấp số nhân gồm 3 số hạng: $tan\frac{5\pi }{12},tan(\frac{5\pi }{12}+\alpha),tan(\frac{5\pi }{12}-\alpha )$.
 
Gọi A là ma trận kề biểu diễn đồ thị G. Kí hiệu $a_{ij}^{(p)}$ là các phần tử của ma trận $A^p=A.A...A$ (p lần). Chứng minh rằng $a_{ij}^{(p)} \; (i,j=1,2,...,n)$ là số các đường đi khác nhau từ đỉnh $i$ đến $j$ độ dài $p$ qua $p-1$ đỉnh trung gian.
 
Cho $(O,R)$ và dây $AB=\sqrt{3}R$. Điểm $M$ di chuyển trên cung lớn $AB$. Gọi $I$ là tâm đường tròn nội tiếp $\Delta AMB$. Đường tròn nội tiếp $\Delta AMB$ tiếp xúc với $MA$ và $MB$ lần lượt tại E và F. Chứng minh $EF$ luôn tiếp xúc với một đường tròn cố định khi $M$ di chuyển trên cung lớn $AB$.
 

Cho $n \in \mathbb{N};n \ge 1$.Ký hiệu $\lim_{n \to +\infty} \left(1+\frac{1}{n} \right)^{n}=e$.Chứng minh rằng:
$$\frac{1}{2ne}<\frac{1}{e}-\left(1-\frac{1}{n} \right)^{n}<\frac{1}{ne}$$

 

$\boxed{\text{Bài toán 344}}$ 

Cho $F_1,F_2,\cdots $ là dãy xác định bởi $F_1=1;F_2=1;F_n=F_{n-1}+F_{n-2}; n\ge 3$. Chứng minh 
$$\sqrt{\frac{F_{n+3}}{F_n}}+\sqrt{\frac{F_n+F_{n+2}}{F_{n+1}}}>1+2\left(\sqrt{\frac{F_n}{F_{n+3}}}+\sqrt{\frac{F_{n+1}}{F_n+F_{n+2}}} \right )$$
 
Tìm tất cả cặp số nguyên dương (a;b) sao cho $ \dfrac{a^b+b}{ab^2+9}$ là một số nguyên
 
Trong phòng rạp có 100 chỗ ngồi và tất cả các vé đã được bán hết (mỗi vé được đánh số thứ tự tương ứng với số chỗ ngồi của phòng rạp).Tìm xác suất để không có khán giả nào ngồi đúng chỗ ghi tên vé của mình
 
$\boxed{\text{Bài toán 347}}$ 
Giả sử rằng $P(x)$ là một đa thức với hệ số thực,có tất cả các nghiệm đều là số ảo.Chứng minh rằng đa thức $P'(x)$ chỉ có một nghiệm thực



#609231 Hãy bố trí các cuộc mitting vào các buổi sao cho số buổi diễn ra là ít nhất

Gửi bởi Ispectorgadget trong 16-01-2016 - 13:09

Giả sử có 6 cuộc mitting A,B,C,D,E,F cần tổ chức. Mỗi cuộc mitting được tổ chức trong một buổi. Các cuộc mitting sau không được diễn đàn đồng thời: BEF,CEF,ABE,CD,AD. Hãy bố trí các cuộc mitting vào các buổi sao cho số buổi diễn ra là ít nhất.